Prove that $f(n)ge 2^{n-1}$












9














This is from a Brazilian math contest for college students (OBMU):



Let $f: (0,+infty) to (0,+infty)$ be a infinitely differentiable function such that




  1. For all positive integer $k$ and positive real $x$, $f^{(k)}(x)> 0$ (where $f^{(k)}$ is the kth derivative).

  2. For all positive integer $m$, $f(m)$ is a positive integer.


Prove that $f(n)ge 2^{n-1}$ for all positive integer $n$.



Attempt



By the mean value theorem, we have



$$f(2)-f(1) = f'(c_1),space c_1 in (1,2)$$
$$f(3)-f(2) = f'(c_2),space c_2 in (2,3)$$
$$vdots $$
$$f(n)-f(n-1) = f'(c_{n-1}),space c_2 in (n,n-1) $$



Then, $f'(c_k)$ is positive integer for all $k in {1,2,cdots, n-1}$. Besides, $f'$ is strictly increasing. Thus, $f'(c_k) ge k$. Adding all the inequalities, we get



$$f(n) ge sum_{k=1}^{n-1}k + f(1) = frac{n(n-1)}{2} + f(1) $$










share|cite|improve this question






















  • Hint: Induct on $n$.
    – user10354138
    Nov 21 '18 at 15:36










  • @user10354138 could you be more specific?
    – Rafael Deiga
    Nov 21 '18 at 17:07
















9














This is from a Brazilian math contest for college students (OBMU):



Let $f: (0,+infty) to (0,+infty)$ be a infinitely differentiable function such that




  1. For all positive integer $k$ and positive real $x$, $f^{(k)}(x)> 0$ (where $f^{(k)}$ is the kth derivative).

  2. For all positive integer $m$, $f(m)$ is a positive integer.


Prove that $f(n)ge 2^{n-1}$ for all positive integer $n$.



Attempt



By the mean value theorem, we have



$$f(2)-f(1) = f'(c_1),space c_1 in (1,2)$$
$$f(3)-f(2) = f'(c_2),space c_2 in (2,3)$$
$$vdots $$
$$f(n)-f(n-1) = f'(c_{n-1}),space c_2 in (n,n-1) $$



Then, $f'(c_k)$ is positive integer for all $k in {1,2,cdots, n-1}$. Besides, $f'$ is strictly increasing. Thus, $f'(c_k) ge k$. Adding all the inequalities, we get



$$f(n) ge sum_{k=1}^{n-1}k + f(1) = frac{n(n-1)}{2} + f(1) $$










share|cite|improve this question






















  • Hint: Induct on $n$.
    – user10354138
    Nov 21 '18 at 15:36










  • @user10354138 could you be more specific?
    – Rafael Deiga
    Nov 21 '18 at 17:07














9












9








9


6





This is from a Brazilian math contest for college students (OBMU):



Let $f: (0,+infty) to (0,+infty)$ be a infinitely differentiable function such that




  1. For all positive integer $k$ and positive real $x$, $f^{(k)}(x)> 0$ (where $f^{(k)}$ is the kth derivative).

  2. For all positive integer $m$, $f(m)$ is a positive integer.


Prove that $f(n)ge 2^{n-1}$ for all positive integer $n$.



Attempt



By the mean value theorem, we have



$$f(2)-f(1) = f'(c_1),space c_1 in (1,2)$$
$$f(3)-f(2) = f'(c_2),space c_2 in (2,3)$$
$$vdots $$
$$f(n)-f(n-1) = f'(c_{n-1}),space c_2 in (n,n-1) $$



Then, $f'(c_k)$ is positive integer for all $k in {1,2,cdots, n-1}$. Besides, $f'$ is strictly increasing. Thus, $f'(c_k) ge k$. Adding all the inequalities, we get



$$f(n) ge sum_{k=1}^{n-1}k + f(1) = frac{n(n-1)}{2} + f(1) $$










share|cite|improve this question













This is from a Brazilian math contest for college students (OBMU):



Let $f: (0,+infty) to (0,+infty)$ be a infinitely differentiable function such that




  1. For all positive integer $k$ and positive real $x$, $f^{(k)}(x)> 0$ (where $f^{(k)}$ is the kth derivative).

  2. For all positive integer $m$, $f(m)$ is a positive integer.


Prove that $f(n)ge 2^{n-1}$ for all positive integer $n$.



Attempt



By the mean value theorem, we have



$$f(2)-f(1) = f'(c_1),space c_1 in (1,2)$$
$$f(3)-f(2) = f'(c_2),space c_2 in (2,3)$$
$$vdots $$
$$f(n)-f(n-1) = f'(c_{n-1}),space c_2 in (n,n-1) $$



Then, $f'(c_k)$ is positive integer for all $k in {1,2,cdots, n-1}$. Besides, $f'$ is strictly increasing. Thus, $f'(c_k) ge k$. Adding all the inequalities, we get



$$f(n) ge sum_{k=1}^{n-1}k + f(1) = frac{n(n-1)}{2} + f(1) $$







real-analysis inequality contest-math






share|cite|improve this question













share|cite|improve this question











share|cite|improve this question




share|cite|improve this question










asked Nov 21 '18 at 2:39









Rafael Deiga

662311




662311












  • Hint: Induct on $n$.
    – user10354138
    Nov 21 '18 at 15:36










  • @user10354138 could you be more specific?
    – Rafael Deiga
    Nov 21 '18 at 17:07


















  • Hint: Induct on $n$.
    – user10354138
    Nov 21 '18 at 15:36










  • @user10354138 could you be more specific?
    – Rafael Deiga
    Nov 21 '18 at 17:07
















Hint: Induct on $n$.
– user10354138
Nov 21 '18 at 15:36




Hint: Induct on $n$.
– user10354138
Nov 21 '18 at 15:36












@user10354138 could you be more specific?
– Rafael Deiga
Nov 21 '18 at 17:07




@user10354138 could you be more specific?
– Rafael Deiga
Nov 21 '18 at 17:07










1 Answer
1






active

oldest

votes


















4














We have
$$f(n+1) = f(n) + int_n^{n+1} f'(t) , d t$$
Since $f(n+1) > f(n)$, because $f'(x) >0$, and $f(n+1)$ and $f(n)$ are integers, we see that $int_n^{n+1} f'(t) , d t$ is an integer. Define now
$$h_1(x) := int_x^{x+1} f'(t) , dt.$$
Note that $h'_1(x) = f'(x+1)-f'(x)$. Also note that $h_1(x)$ is integer-valued on $mathbb{N}$ and has the property that $h^{(i)}_1(x) >0$, i.e. $h_1$ has the same properties as $f$.



We can proceed in this manner to obtain functions $h_n$ with the properties (1) and (2). (For example, apply the same argument on $h_1$ instead of $f$ in order to get $h_2$.) This construction satisfies $$h_i(n+1) = h_i(n) + h_{i+1}(n).$$
We prove by induction that $h_i(j) ge 2^{j-1}$ for all $i in mathbb{N}_0$, whereby we define $h_0 =f$, simultaneously.




  1. If $j=1$ then $h_i(1)$ is an integer (for all $i in mathbb{N}_0$) and thus the lower bound is satisfied.

  2. Assume that the statement was already proven for all $ 1 le j le n$ and $i in mathbb{N}_0$. Then we have
    $$h_i(n+1) = h_i(n) + h_{i+1}(n) ge 2^{n-1} + 2^{n-1} = 2^{n}.$$


As a special case, we get $f(n) = h_0(n) ge 2^{n-1}$.






share|cite|improve this answer























    Your Answer





    StackExchange.ifUsing("editor", function () {
    return StackExchange.using("mathjaxEditing", function () {
    StackExchange.MarkdownEditor.creationCallbacks.add(function (editor, postfix) {
    StackExchange.mathjaxEditing.prepareWmdForMathJax(editor, postfix, [["$", "$"], ["\\(","\\)"]]);
    });
    });
    }, "mathjax-editing");

    StackExchange.ready(function() {
    var channelOptions = {
    tags: "".split(" "),
    id: "69"
    };
    initTagRenderer("".split(" "), "".split(" "), channelOptions);

    StackExchange.using("externalEditor", function() {
    // Have to fire editor after snippets, if snippets enabled
    if (StackExchange.settings.snippets.snippetsEnabled) {
    StackExchange.using("snippets", function() {
    createEditor();
    });
    }
    else {
    createEditor();
    }
    });

    function createEditor() {
    StackExchange.prepareEditor({
    heartbeatType: 'answer',
    autoActivateHeartbeat: false,
    convertImagesToLinks: true,
    noModals: true,
    showLowRepImageUploadWarning: true,
    reputationToPostImages: 10,
    bindNavPrevention: true,
    postfix: "",
    imageUploader: {
    brandingHtml: "Powered by u003ca class="icon-imgur-white" href="https://imgur.com/"u003eu003c/au003e",
    contentPolicyHtml: "User contributions licensed under u003ca href="https://creativecommons.org/licenses/by-sa/3.0/"u003ecc by-sa 3.0 with attribution requiredu003c/au003e u003ca href="https://stackoverflow.com/legal/content-policy"u003e(content policy)u003c/au003e",
    allowUrls: true
    },
    noCode: true, onDemand: true,
    discardSelector: ".discard-answer"
    ,immediatelyShowMarkdownHelp:true
    });


    }
    });














    draft saved

    draft discarded


















    StackExchange.ready(
    function () {
    StackExchange.openid.initPostLogin('.new-post-login', 'https%3a%2f%2fmath.stackexchange.com%2fquestions%2f3007181%2fprove-that-fn-ge-2n-1%23new-answer', 'question_page');
    }
    );

    Post as a guest















    Required, but never shown

























    1 Answer
    1






    active

    oldest

    votes








    1 Answer
    1






    active

    oldest

    votes









    active

    oldest

    votes






    active

    oldest

    votes









    4














    We have
    $$f(n+1) = f(n) + int_n^{n+1} f'(t) , d t$$
    Since $f(n+1) > f(n)$, because $f'(x) >0$, and $f(n+1)$ and $f(n)$ are integers, we see that $int_n^{n+1} f'(t) , d t$ is an integer. Define now
    $$h_1(x) := int_x^{x+1} f'(t) , dt.$$
    Note that $h'_1(x) = f'(x+1)-f'(x)$. Also note that $h_1(x)$ is integer-valued on $mathbb{N}$ and has the property that $h^{(i)}_1(x) >0$, i.e. $h_1$ has the same properties as $f$.



    We can proceed in this manner to obtain functions $h_n$ with the properties (1) and (2). (For example, apply the same argument on $h_1$ instead of $f$ in order to get $h_2$.) This construction satisfies $$h_i(n+1) = h_i(n) + h_{i+1}(n).$$
    We prove by induction that $h_i(j) ge 2^{j-1}$ for all $i in mathbb{N}_0$, whereby we define $h_0 =f$, simultaneously.




    1. If $j=1$ then $h_i(1)$ is an integer (for all $i in mathbb{N}_0$) and thus the lower bound is satisfied.

    2. Assume that the statement was already proven for all $ 1 le j le n$ and $i in mathbb{N}_0$. Then we have
      $$h_i(n+1) = h_i(n) + h_{i+1}(n) ge 2^{n-1} + 2^{n-1} = 2^{n}.$$


    As a special case, we get $f(n) = h_0(n) ge 2^{n-1}$.






    share|cite|improve this answer




























      4














      We have
      $$f(n+1) = f(n) + int_n^{n+1} f'(t) , d t$$
      Since $f(n+1) > f(n)$, because $f'(x) >0$, and $f(n+1)$ and $f(n)$ are integers, we see that $int_n^{n+1} f'(t) , d t$ is an integer. Define now
      $$h_1(x) := int_x^{x+1} f'(t) , dt.$$
      Note that $h'_1(x) = f'(x+1)-f'(x)$. Also note that $h_1(x)$ is integer-valued on $mathbb{N}$ and has the property that $h^{(i)}_1(x) >0$, i.e. $h_1$ has the same properties as $f$.



      We can proceed in this manner to obtain functions $h_n$ with the properties (1) and (2). (For example, apply the same argument on $h_1$ instead of $f$ in order to get $h_2$.) This construction satisfies $$h_i(n+1) = h_i(n) + h_{i+1}(n).$$
      We prove by induction that $h_i(j) ge 2^{j-1}$ for all $i in mathbb{N}_0$, whereby we define $h_0 =f$, simultaneously.




      1. If $j=1$ then $h_i(1)$ is an integer (for all $i in mathbb{N}_0$) and thus the lower bound is satisfied.

      2. Assume that the statement was already proven for all $ 1 le j le n$ and $i in mathbb{N}_0$. Then we have
        $$h_i(n+1) = h_i(n) + h_{i+1}(n) ge 2^{n-1} + 2^{n-1} = 2^{n}.$$


      As a special case, we get $f(n) = h_0(n) ge 2^{n-1}$.






      share|cite|improve this answer


























        4












        4








        4






        We have
        $$f(n+1) = f(n) + int_n^{n+1} f'(t) , d t$$
        Since $f(n+1) > f(n)$, because $f'(x) >0$, and $f(n+1)$ and $f(n)$ are integers, we see that $int_n^{n+1} f'(t) , d t$ is an integer. Define now
        $$h_1(x) := int_x^{x+1} f'(t) , dt.$$
        Note that $h'_1(x) = f'(x+1)-f'(x)$. Also note that $h_1(x)$ is integer-valued on $mathbb{N}$ and has the property that $h^{(i)}_1(x) >0$, i.e. $h_1$ has the same properties as $f$.



        We can proceed in this manner to obtain functions $h_n$ with the properties (1) and (2). (For example, apply the same argument on $h_1$ instead of $f$ in order to get $h_2$.) This construction satisfies $$h_i(n+1) = h_i(n) + h_{i+1}(n).$$
        We prove by induction that $h_i(j) ge 2^{j-1}$ for all $i in mathbb{N}_0$, whereby we define $h_0 =f$, simultaneously.




        1. If $j=1$ then $h_i(1)$ is an integer (for all $i in mathbb{N}_0$) and thus the lower bound is satisfied.

        2. Assume that the statement was already proven for all $ 1 le j le n$ and $i in mathbb{N}_0$. Then we have
          $$h_i(n+1) = h_i(n) + h_{i+1}(n) ge 2^{n-1} + 2^{n-1} = 2^{n}.$$


        As a special case, we get $f(n) = h_0(n) ge 2^{n-1}$.






        share|cite|improve this answer














        We have
        $$f(n+1) = f(n) + int_n^{n+1} f'(t) , d t$$
        Since $f(n+1) > f(n)$, because $f'(x) >0$, and $f(n+1)$ and $f(n)$ are integers, we see that $int_n^{n+1} f'(t) , d t$ is an integer. Define now
        $$h_1(x) := int_x^{x+1} f'(t) , dt.$$
        Note that $h'_1(x) = f'(x+1)-f'(x)$. Also note that $h_1(x)$ is integer-valued on $mathbb{N}$ and has the property that $h^{(i)}_1(x) >0$, i.e. $h_1$ has the same properties as $f$.



        We can proceed in this manner to obtain functions $h_n$ with the properties (1) and (2). (For example, apply the same argument on $h_1$ instead of $f$ in order to get $h_2$.) This construction satisfies $$h_i(n+1) = h_i(n) + h_{i+1}(n).$$
        We prove by induction that $h_i(j) ge 2^{j-1}$ for all $i in mathbb{N}_0$, whereby we define $h_0 =f$, simultaneously.




        1. If $j=1$ then $h_i(1)$ is an integer (for all $i in mathbb{N}_0$) and thus the lower bound is satisfied.

        2. Assume that the statement was already proven for all $ 1 le j le n$ and $i in mathbb{N}_0$. Then we have
          $$h_i(n+1) = h_i(n) + h_{i+1}(n) ge 2^{n-1} + 2^{n-1} = 2^{n}.$$


        As a special case, we get $f(n) = h_0(n) ge 2^{n-1}$.







        share|cite|improve this answer














        share|cite|improve this answer



        share|cite|improve this answer








        edited Nov 22 '18 at 9:58

























        answered Nov 22 '18 at 9:39









        p4sch

        4,770217




        4,770217






























            draft saved

            draft discarded




















































            Thanks for contributing an answer to Mathematics Stack Exchange!


            • Please be sure to answer the question. Provide details and share your research!

            But avoid



            • Asking for help, clarification, or responding to other answers.

            • Making statements based on opinion; back them up with references or personal experience.


            Use MathJax to format equations. MathJax reference.


            To learn more, see our tips on writing great answers.





            Some of your past answers have not been well-received, and you're in danger of being blocked from answering.


            Please pay close attention to the following guidance:


            • Please be sure to answer the question. Provide details and share your research!

            But avoid



            • Asking for help, clarification, or responding to other answers.

            • Making statements based on opinion; back them up with references or personal experience.


            To learn more, see our tips on writing great answers.




            draft saved


            draft discarded














            StackExchange.ready(
            function () {
            StackExchange.openid.initPostLogin('.new-post-login', 'https%3a%2f%2fmath.stackexchange.com%2fquestions%2f3007181%2fprove-that-fn-ge-2n-1%23new-answer', 'question_page');
            }
            );

            Post as a guest















            Required, but never shown





















































            Required, but never shown














            Required, but never shown












            Required, but never shown







            Required, but never shown

































            Required, but never shown














            Required, but never shown












            Required, but never shown







            Required, but never shown







            Popular posts from this blog

            Can a sorcerer learn a 5th-level spell early by creating spell slots using the Font of Magic feature?

            Does disintegrating a polymorphed enemy still kill it after the 2018 errata?

            A Topological Invariant for $pi_3(U(n))$